15 votes
Accepted

Upper bound for Hall's conjecture on separation of squares and cubes

The best $\theta$ is $0$. It is known that there are infinitely many solutions of 0 < $|x^3 - y^2| \ll x^{1/2}$, parametrized by certain "Pell equations"; indeed one such family attains $...
Noam D. Elkies's user avatar
13 votes
Accepted

Continuous variant of the Chinese remainder theorem

Your guess is correct. For $\alpha = (\alpha_1, \alpha_2, \ldots, \alpha_d) \in \mathbb{R}^d$, the values of $m \alpha \bmod 1$ are equidistributed (and in particular dense) in $(\mathbb{R}/\mathbb{Z})...
David E Speyer's user avatar
10 votes
Accepted

Integral points near elliptic curves

You can take $\theta = 0$, even $\theta = -1/6$ works. Fix an integer $A \ne 0$ and an integer $B$. If $r$ is an integer, the elliptic curve $E : y^{2} = x^{3} + Ax + r^{2} A^{2}$ has the obvious ...
Jeremy Rouse's user avatar
7 votes
Accepted

Central limit theorem for irrational rotations

I am surprised by the question. The sums are bounded if $\alpha \ne 1$, since for every $n \ge 1$, $$\Big|\sum_{k=1}^n \Re R_\alpha^k z \Big| = \Big|\Re \Big(\sum_{k=1}^n R_\alpha^k z \Big) \Big| = \...
Christophe Leuridan's user avatar
6 votes
Accepted

The liminf of an expression involving an irrational rotation

The answer is no, because for some real $c>0$ and all integers $q>0$ we have $$q\{q\sqrt2\}=q(q\sqrt2-\lfloor q\sqrt2\rfloor) =q|q\sqrt2-\lfloor q\sqrt2\rfloor| \\ \ge q\,\inf_{p\in\mathbb Z}|q\...
Iosif Pinelis's user avatar
4 votes
Accepted

Diophantine equations involving recurrence sequences

The authors skipped some easy steps. By the triangle inequality, $$e^{z_1}\geq 1-|1-e^{z_1}|>1-0.95=1/20,$$ whence using also $z_1<0$, $$e^{|z_1|}=e^{-z_1}<20.$$ Similarly, $$e^{z_2}\geq 1-|1-...
GH from MO's user avatar
  • 99.2k
3 votes
Accepted

Simultaneously approximating all $x \in [0,1]$ with fractions of bounded denominator

Fix $T$ ($T=100$ in your example) and denote by $S_T(n)$ the set of numbers $x\in [0,1]$ such that $|x-p/q|>1/n^2$ whenever $p,q$ are integers and $1\leqslant q\leqslant Tn$ (so, my $n$ is your $\...
Fedor Petrov's user avatar
3 votes
Accepted

Simultaneous rational approximations of multiples of the golden ratio

Note that since you want $|n_m \alpha| \to 0$ this implies that for almost all $m$ you have $|n_m \alpha| < \frac{1}{2k}$, and in that case you have $ |n_m k \alpha | = k |n_m \alpha |$. You can ...
Command Master's user avatar
3 votes
Accepted

Extreme case bounds on Diophantine approximation

If $\alpha$ is a real irrational number, then there are infinitely many coprime integers $p,q$ with $q > 0$ such that $$\displaystyle \left \lvert \alpha - \frac{p}{q} \right \rvert < \frac{1}{q^...
Stanley Yao Xiao's user avatar
1 vote

Diophantine equations involving the difference between perfect square and perfect cube

Just to point out that this is related to integral points on elliptic curves, which is well studied topic. Treating $z$ as parameter, your equation (a) is equivalent to integral points on the elliptic ...
joro's user avatar
  • 24.2k

Only top scored, non community-wiki answers of a minimum length are eligible